Đến nội dung

Hình ảnh

Hơi khó với cấp 2


  • Please log in to reply
Chủ đề này có 39 trả lời

#21
NPKhánh

NPKhánh

    Tiến sĩ toán

  • Thành viên
  • 1115 Bài viết
Cho x;y;z thoả mãn $x^2+ y^2+ z^2 = 9$. Tìm $max f(x;y;z) = xy^3+yz^3+ zx^3 =?$

http://mathsvn.violet.vn trang ebooks tổng hợp miễn phí , nhiều tài liệu ôn thi Đại học



http://www.maths.vn Diễn đàn tổng hợp toán -lý - hóa ... dành cho học sinh THCS ;THPT và Sinh viên


#22
supermember

supermember

    Đại úy

  • Hiệp sỹ
  • 1646 Bài viết

Cho x;y;z thoả mãn $x^2+ y^2+ z^2 = 9$. Tìm $max f(x;y;z) = xy^3+yz^3+ zx^3 =?$

Bài này đưa về 1 bài của Vasc:CMR $3(xy^3+yz^3+ zx^3) \leq (x^2+y^2+z^2)^2 $
Khi bạn là người yêu Toán, hãy chấp nhận rằng bạn sẽ buồn nhiều hơn vui :)

#23
dtdong91

dtdong91

    Tiến sĩ diễn đàn toán

  • Hiệp sỹ
  • 1791 Bài viết
He thầy post ở đây rùi mà
http://diendantoanho...showtopic=29134
12A1-THPT PHAN BỘI CHÂU-TP VINH-NGHỆ AN

SẼ LUÔN LUÔN Ở BÊN BẠN

#24
daingu

daingu

    Lính mới

  • Thành viên
  • 3 Bài viết
Cho: (x+y+z)xyz=1
Tìm gtnn của M=(x+y)(x+z)


#25
xvodanhx

xvodanhx

    Binh nhất

  • Thành viên
  • 31 Bài viết
Bài này có gì đâu khó
GIẢI
ta có: (x+y+z)xyz =1
<=> $\ x^{2} $ + $\ y^{2} $ + $\ z^{2} $ + 2xy+2yz+2xz =1
<=> $\ (x+y+z)^{3} $ =1
<=> x+y+z=1
=> xyz=1
.Mặt khác: M =(x+y)(x+z)=(1-z)(1-y)= 1-z-y +yz = x + $\dfrac{1}{x} $
...........
TỚI Đó CHẮC AI CŨNG LÀM DC RỒI !!!!!!!!

Bài viết đã được chỉnh sửa nội dung bởi xvodanhx: 25-02-2007 - 11:30


#26
daingu

daingu

    Lính mới

  • Thành viên
  • 3 Bài viết
(x+y+z)xyz=1 <=> x2+y2+z2+2xy+2yz+2zx =1 ?

Bài viết đã được chỉnh sửa nội dung bởi daingu: 25-02-2007 - 13:34


#27
dtdong91

dtdong91

    Tiến sĩ diễn đàn toán

  • Hiệp sỹ
  • 1791 Bài viết
Hì dùng AM-GM thui
xyz(x+y+z)=$ yz(x^2+xy+xz) \leq \dfrac{(x^2+xy+yz+zx)^2}{4} =\dfrac{[(x+y)(x+z)]^2}{4}$
Xong xuôi
12A1-THPT PHAN BỘI CHÂU-TP VINH-NGHỆ AN

SẼ LUÔN LUÔN Ở BÊN BẠN

#28
10maths_tp0609

10maths_tp0609

    Zarai Nakeda XIII

  • Thành viên
  • 218 Bài viết
$ M^2 \geq 4 \rightarrow min M= ???? $
bài này có đk a,b,c :D 0 không vậy.

Bài viết đã được chỉnh sửa nội dung bởi 10maths_tp0609: 25-02-2007 - 18:32

Zarai "từ cấm"a XIII

#29
chien than

chien than

    Thượng sĩ

  • Thành viên
  • 272 Bài viết
Ta có:
$a_1^2=1-b_1^2=(a_1^2+a_2^2+a_3^2+a_4^2+a_5^2)$
=>$b_1=\sqrt{a_2^2+a_3^2+a_4^2+a_5^2} \geq \dfrac{1}{2}(a_2+a_3+a_4+a_5)$
Tương tự tự có:
$b_2 \geq \dfrac{1}{2}(a_1+a_3+a_4+a_5)$
$b_3 \geq \dfrac{1}{2}(a_1+a_2+a_4+a_5)$
$b_4 \geq \dfrac{1}{2}(a_1+a_2+a_3+a_5)$
$b_5 \geq \dfrac{1}{2}(a_1+a_2+a_3+a_4}$
Cộng từng vế các BĐT trên được:
$b_1+b_2+b_3+b_4+b_5 \geq 2(a_1+a_2+a_3+a_4+a_5)$
=>$\dfrac{b_1+b_2+...+b_5}{a_1+a_2+...+a_5} \geq 2$

#30
dtdong91

dtdong91

    Tiến sĩ diễn đàn toán

  • Hiệp sỹ
  • 1791 Bài viết
Hừm hôm trước tui đưa đường link rùi mà
Cậu Vũ đã post ở đay rùi
http://diendantoanho...showtopic=28744
12A1-THPT PHAN BỘI CHÂU-TP VINH-NGHỆ AN

SẼ LUÔN LUÔN Ở BÊN BẠN

#31
Cindy_cute

Cindy_cute

    No Deja Vu just me n you

  • Thành viên
  • 78 Bài viết
thử giải bài này đi mấy bạn ...
Cho a,b,c > $ \sqrt[3]{ \dfrac{1}{4} } $
Cmr:
$ \dfrac{a+b+c}{4(2\sqrt{abc}-1)} \geq \dfrac{1}{(b+c)^2} +\dfrac{1}{(c+a)^2} + \dfrac{1}{(a+b)^2} $

Bài viết đã được chỉnh sửa nội dung bởi Cindy_cute: 28-02-2007 - 21:45

So... we've done!!!.

#32
dtdong91

dtdong91

    Tiến sĩ diễn đàn toán

  • Hiệp sỹ
  • 1791 Bài viết
Hix thường thì đối với những bài này sai đề là rất dễ :D
Bài này cũng ko phải là ngoại lệ
Cho $ a=b=c=\sqrt[3]{\dfrac{1}{3}}$

Bài viết đã được chỉnh sửa nội dung bởi dtdong91: 01-03-2007 - 20:47

12A1-THPT PHAN BỘI CHÂU-TP VINH-NGHỆ AN

SẼ LUÔN LUÔN Ở BÊN BẠN

#33
Cindy_cute

Cindy_cute

    No Deja Vu just me n you

  • Thành viên
  • 78 Bài viết

Hix thường thì đối với những bài này sai đề là rất dễ :D
Bài này cũng ko phải là ngoại lệ
Cho $ a=b=c =\sqrt[3]{\dfrac{1}{3}}$


Trùi ui thế lại dùm đi bạn, hoàn toàn đúng bất đẳng thức

Nếu thế số như bạn thì VP = khoảng 3.3614683 còn VT = khoảng 1.56 hê hế sao mà sai được
So... we've done!!!.

#34
mathmath

mathmath

    tuổi trẻ -những nẻo đường tương lai

  • Thành viên
  • 288 Bài viết
$\dfrac{1}{(a+b)^2}\leq\dfrac{1}{4ab}$ quy đồng suy ra bdt tương đương
$abc+1\geq\2sqrt{abc} $ "=" khi a=b=c=1

Bài viết đã được chỉnh sửa nội dung bởi mathmath: 02-03-2007 - 17:34

VMF my love!!! Bye Math :(( Bye VMF :(( sì u ờ gên hihi ^^

#35
dtdong91

dtdong91

    Tiến sĩ diễn đàn toán

  • Hiệp sỹ
  • 1791 Bài viết
Hì hôm qua anh ốm mà hôm nay lên đã thấy chú em giải mất rùi
Thui đành góp thêm một cách tru bò vậy :D
Xét a :D b : geq c
=> $ f(a,b,c) \geq f(\sqrt{ab},\sqrt{ab},c)$
Cái này đúng do $ \dfrac{2}{(\sqrt{ab}+c)^2} \geq \dfrac{1}{(a+c)^2}+\dfrac{1}{(b+c)^2}$
12A1-THPT PHAN BỘI CHÂU-TP VINH-NGHỆ AN

SẼ LUÔN LUÔN Ở BÊN BẠN

#36
MyLoveIs4Ever

MyLoveIs4Ever

    Sĩ quan

  • Thành viên
  • 441 Bài viết
Cho a,b,c dương.CMR:
$\large\ 2^6\dfrac{abcd+1}{(a+b+c+d)^2} \leq a^2+b^2+c^2+d^2+\dfrac{1}{a^2}+\dfrac{1}{b^2}+\dfrac{1}{c^2}+\dfrac{1}{d^2}$

#37
supermember

supermember

    Đại úy

  • Hiệp sỹ
  • 1646 Bài viết
C/m:$\sum a^2 \geq 2(ab+bc+ca-6)$ với a,b,c>0 thỏa mãn abc=8

Bài viết đã được chỉnh sửa nội dung bởi supermember: 07-03-2007 - 19:17

Khi bạn là người yêu Toán, hãy chấp nhận rằng bạn sẽ buồn nhiều hơn vui :)

#38
dtdong91

dtdong91

    Tiến sĩ diễn đàn toán

  • Hiệp sỹ
  • 1791 Bài viết
Đúng là dễ thiệt :D
C1 : Dùng dồn biến
f(a,b,c) :leq $ f(\sqrt{ab},\sqrt{ab},c)$
C2 : Dùng tam thức bậc 2
12A1-THPT PHAN BỘI CHÂU-TP VINH-NGHỆ AN

SẼ LUÔN LUÔN Ở BÊN BẠN

#39
10maths_tp0609

10maths_tp0609

    Zarai Nakeda XIII

  • Thành viên
  • 218 Bài viết
$ 4(\dfrac{1}{a^2}+\dfrac{1}{b^2}+\dfrac{1}{c^2}+\dfrac{1}{d^2}) \geq (\dfrac{1}{a}+\dfrac{1}{b}+\dfrac{1}{c}+\dfrac{1}{d})^2 \geq \dfrac{16^2}{(a+b+c+a)^2} $
$ (a^2+b^2+c^2+d^2)(a+b+c+d)^2 \geq 2^6abcd $

Bài viết đã được chỉnh sửa nội dung bởi 10maths_tp0609: 08-03-2007 - 19:53

Zarai "từ cấm"a XIII

#40
dtdong91

dtdong91

    Tiến sĩ diễn đàn toán

  • Hiệp sỹ
  • 1791 Bài viết
Còn cách giải khác cũng khá là dùng bổ đề của anh VQBC
Với a+b+c=p,ab+bc+ca=$ \dfrac{p^2-q^2}{3}$
=>$\dfrac{(p+q)^2(p-2q)}{27} \leq abc \leq \dfrac{(p-q)^2(p+2q)}{27}$
12A1-THPT PHAN BỘI CHÂU-TP VINH-NGHỆ AN

SẼ LUÔN LUÔN Ở BÊN BẠN




1 người đang xem chủ đề

0 thành viên, 1 khách, 0 thành viên ẩn danh